Differential equation dv/dt = 9.8 - v/5, v(0) = 0

Click For Summary
The discussion revolves around solving the initial value problem for a falling object described by the differential equation dv/dt = 9.8 - v/5, with the initial condition v(0) = 0. The time required for the object to reach 98% of its limiting velocity is calculated to be t = 19.56 seconds, with the velocity function given as v(t) = -49e^(-t/5) + 49. However, there are discrepancies in calculating the distance fallen during this time, with the expected distance being 718.34. Participants note that the integration process may have errors due to incorrect limits, and emphasize the importance of evaluating the definite integral correctly to find the distance fallen. The discussion highlights the need for careful handling of integration constants to ensure accurate results.
e^(i Pi)+1=0
Messages
246
Reaction score
1
A falling object satisfies the initial value problem:

dv/dt = 9.8 - v/5, v(0) = 0

1.Find the time that must elapse for the object to reach 98% of its limiting velocity.

answer: t = 19.56, and for completeness, v = -49e-t/5 + 49

2.How far does the object fall in the time found in part a?

Integrating yields the wrong answer, which should = 718.34?

p(t) = 245e-t/5 + 49t
 
Physics news on Phys.org
e^(i Pi)+1=0 said:
A falling object satisfies the initial value problem:

dv/dt = 9.8 - v/5, v(0) = 0

1.Find the time that must elapse for the object to reach 98% of its limiting velocity.

answer: t = 19.56, and for completeness, v = -49e-t/5 + 49

2.How far does the object fall in the time found in part a?

Integrating yields the wrong answer, which should = 718.34?

p(t) = 245e-t/5 + 49t
What was it you integrated, and how did you do it?
 
SammyS said:
What was it you integrated, and how did you do it?
The p(t) at the bottom of the OP is suggests he/she integrated v(t) from part 1. Must be a problem with the limits of integration. I've gotten the right answer, and double-checked it.
 
SithsNGiggles said:
The p(t) at the bottom of the OP is suggests he/she integrated v(t) from part 1. Must be a problem with the limits of integration. I've gotten the right answer, and double-checked it.
Fair enough !

To OP:

What is p(19.56) - p(0) ?
 
It was pointed out to me elsewhere that was how to do it. I did an indefinite integral and then evaluated p(19.56). I'm having a difficult time intuiting why that would yield a wrong answer though.
 
e^(i Pi)+1=0 said:
It was pointed out to me elsewhere that was how to do it. I did an indefinite integral and then evaluated p(19.56). I'm having a difficult time intuiting why that would yield a wrong answer though.
Because for, p(t), as you have it, p(0) ≠ 0 . You could have used a constant of integration to make p(0) = 0 .
 
Question: A clock's minute hand has length 4 and its hour hand has length 3. What is the distance between the tips at the moment when it is increasing most rapidly?(Putnam Exam Question) Answer: Making assumption that both the hands moves at constant angular velocities, the answer is ## \sqrt{7} .## But don't you think this assumption is somewhat doubtful and wrong?

Similar threads

  • · Replies 2 ·
Replies
2
Views
2K
Replies
3
Views
2K
Replies
3
Views
2K
  • · Replies 12 ·
Replies
12
Views
2K
Replies
7
Views
2K
Replies
32
Views
3K
  • · Replies 10 ·
Replies
10
Views
6K
  • · Replies 1 ·
Replies
1
Views
2K
  • · Replies 20 ·
Replies
20
Views
5K
Replies
3
Views
2K